BCPS practice problems

Pataasin ang iyong marka sa homework at exams ngayon gamit ang Quizwiz!

Which guidance should be given to a patient receiving amphetamine/dextroamphetamine mixed salts extended release 20 mg orally once daily for a recent diagnosis of attention deficit hyperactivity disorder? - drowsiness is possible - suicidial ideation - weight gain -tic developement

Suicide

B.D. is a 73-year-old man (height 69 inches, weight 80 kg) with newly diagnosed NVAF. He also has a history of hypertension, dyslipidemia, stable ischemic heart disease, and systolic heart failure. His current medications include aspirin 81 mg daily, enalapril 10 mg daily, atorvastatin 80 mg daily, metoprolol succinate 200 mg daily, furosemide 40 mg daily, spironolactone 25 mg daily, and amlodipine 10 mg daily. His current heart rate is 72 beats/ minute and blood pressure is 122/72 mm Hg. His laboratory values include potassium 4.9 mEq/L, stable SCr 1.9 mEq/L, and blood glucose 101 mg/dL. 1. Which best depicts B.D.'s CHA2 DS2 -VASc score? A. 2. B. 3. C. 4. D. 5 Which is the most appropriate regimen for reducing B.D.'s risk of stroke? A. Dabigatran 75 mg twice daily. B. Rivaroxaban 20 mg once daily. C. Apixaban 5 mg twice daily. D. Edoxaban 60 mg once daily.

3 CHF HTN Age under 75 but over 65 SIHD doesnt count unless it was an MI, PAD or aortic disease Sc= sex category(f) NOT Scr C= does not meet the 2 of 3 criteria for lowering dose in AFIB crcl=40 A= doesnt get dose decreased till <30 B= under 50 so should be 15mg daily D= should be 30mg daily

Which patient is a candidate for the 12-dose directly observed therapy regimen of isoniazid/rifapentine for latent tuberculosis infection? - 6mo infant -26 yo man HIV + and on HART - 36yo pregnant woman - 40 yo man with HTN

40 yr old non HIV pts only can do it one that has higher risk of non compliance

DES following elective PCI, how long is DAPT for? 1 mo 3mo 6mo 12mo

6 mo not in setting ACS as this is ELECTIVE. if ACS this is 12months no matter the stent(unless high bleed risk or other issues like mortality risk) 1mo is for Bare metal stent

In a study where rivaroxaban was compared to enoxaparin to find total venous thromboembolism(VTE) following hip replacement surgery, there were 17 total VTE out of 1513 patients in the rivaraoaban group and 57 total VTE out of 1473 patient in the enoxaparin group. What is the relative risk reduction of using rivaroxaban over enoxaparin?

71% Relative risk: (Event rate in rivaroxaban group)/(Event rate in enoxaparin group) = (17/1513)/(57/1473) = 0.2903Relative risk reduction: 1 - (relative risk) = 1 - 0.2903 = 0.7097 = 0.71

A 70-year-old woman. She has complaints of fatigue, light-headedness, constipation, and "too many medicines." PMH: HTN, CAD(DES 8 years ago), COPD, DM, incontinence, frequent UTIs, depression, and moderate dementia. Vitals: BP 160/82 mm Hg, HR 51 beats/minute, RR: 16 O2 sat 99% on room air. Her current medications are as follows: Advair, ASA 81mg, APAP 650 TID, Plavix daily, donepezil 10 mg daily, glipizide 5 mg BID, lisinopril 10 mg daily, loratadine, metoprolol 50 mg BID, paroxetine 40 mg daily, ranitidine 150 mg BID, simvastatin 40 mg HS, and tolterodine 2 mg HS. Macrobid 100 mg twice daily for 10 days was initiated 3 days ago. Labs from 3 days ago: Na 130 mg/dL, CO2 24 mEq/dL, BUN 24 mg/dL, SCr 1.6 mg/dL, FBG 67 mg/dL, A1C 6.3%, UA: Positive for UTI Which medication list best depicts the medications with the greatest potential to harm this patient, according to the AGS 2019 Beers Criteria? A. Paxil, donepezil, detrol. B. Donepezil, glipizide, Zocor. C. Glipizide, donepezil, macrobide. D. toprol, plavix, zantac.

A

A 68YOM w/ hx of stage 5 CKD on HD, HTN, CAD post-MI, moderately depressed LVEF, and GERD presents with acute-onset SOB and chest pain. After his recent dialysis, he was nonadherent to med x2 days and noticed he had gained 2 kg in 24 hours. His baseline orthopnea worsened to sleeping sitting up in a chair for the 2 nights before admission. He admits smoking cocaine within the past 24 hours and developed acute-onset chest tightness with diaphoresis and nausea, and his pain was 7/10. He went to the ED, with a BP:250/120 mm Hg. He had crackles halfway up his lungs on examination, and chest radiography detected bilateral fluffy infiltrates. His ECG: sinus tach, HR 122 BPM, and ST-segment depressions in leads 2, 3, and aVF. He was admitted for a HTN emergency. Labs: BUN 48 mg/dL, SCr 11.4 mg/dL, BNP 2350 pg/mL, troponin T 1.5 ng/L (less than 0.1 mcg/L), CK 227 units/L, and CK-MB 22 units/L. Which med is best for this patient's HTN emergency: A. IV NTG titrated to a 25% reduction in MAP. B. Labetalol titrated to a 50% reduction in MAP. C.NTP titrated to a 25% reduction in MAP. D. Clonidine PO 50% reduction in MAP.

A MAP decrease is 25% over 1st hr then go to 160/100 over the next 2-6 hours B=BB use is controversial if hx of cocaine use. 50% reduction is not appropriate C=renal impaiment so NTG>NTP due to thiocyanat tox D=difficult to titrate oral dosage forms and could go over the 25% decrease

A 66-year-old man with a medical history of HTN and acute coronary syndrome with a drug-eluting coronary stent placement 14 months ago presents to the primary care clinic. Current medications include aspirin 81 mg/day, prasugrel 10 mg/day, nitroglycerin 0.4-mg sublingual tablets as needed for chest pain, metoprolol succinate 75 mg/day, ramipril 10 mg/day, and atorvastatin 20 mg/day. He asks you how long he will need to take prasugrel. What is the best answer? A. Call your physician because you may be able to stop prasugrel now. B. Your prasugrel should have been discontinued 6 months after acute coronary syndrome; discontinue it now. C. You will need to take prasugrel indefinitely. D. You will need to take prasugrel for at least 18 months after your MI and stent placement.

A P2Y12 is for 12months in most pts B= 6 months of DAPT is onlyf or those with high bleed risk C=DAPT indef is not supported D=12 is minimum not 18mo

A 72-year-old man is admitted to the hospital for HF decompensation. The patient has progressively increased dyspnea when walking and orthopnea increased bilateral lower-extremity swelling, 13 kg of weight gain in the past 3 weeks, and dietary nonadherence. He has a hx of(LVEF 25%, NYHA class III), paroxysmal AF, and hyperlipidemia. Pertinent laboratory values are as follows: BNP 2300 pg/mL (0-50 pg/mL), K+ 4.9 mEq/L, BUN 32 mg/dL, SCr 2.0 mg/dL (baseline 1.9 mg/dL),BP 108/62mm Hg, HR 82BPM, and Sao2 95%. Home medications include carvedilol, lisinopril, furosemide, spironolactone, and digoxin. The patient initially responds with 2 L of urinary output but then decreased. SCr is now 4.3 mg/dL. Is drowsy, confused, cyanotic and cool to the touch. BP 89/68 and HR 98. CI =1.5, SVR 2650, PCWP=30 Best tx: A. Milrinone 0.2 mcg/kg/minute. B. Dobutamine 10 mcg/kg/minute. C. Sodium nitroprusside 0.1 mcg/kg/minute. D. Phenylephrine 20 mcg/minute.

A cold and wet state milrinone will increase CO while also lower SVR Dobutamine is not recommeded if pt has BB therapy and this dose is aggressive NTP relative CI in pts with SBP less than 100 and CI in SBP < 90 PE has no positive beta effects and ther fore will not augment contracility. alpha stimulation will cause SVR to increase and CO to further decrease

R.P. is an 69-year-old African American man with a history of HTN and gout. His medications include allopurinol 300 mg/day, amlodipine 10 mg/day, and aspirin 81 mg/day. His vital signs include BP 145/85 mm Hg and HR 82 beats/minute. His laboratory values are normal and his 10-year ASCVD risk is 22.4%. Which is the best therapy for R.P.? A. Add hydrochlorothiazide 25 mg/day to achieve a systolic blood pressure (SBP) goal of less than 130 mm Hg. B. Add lisinopril 40 mg/day and titrate to achieve an SBP goal of less than 140. C. Add atenolol 50 mg/day to achieve an SBP less than 130 mm Hg. D. Make no changes to his current medications because his SBP is at goal.

A goal is 130/80 bc no CVD but has 10yr ASCVD >10%= B and D are wrong BB is okay if pt has CAD, HF or aFib

A 72YOM is admitted to the hospital for HF decomp. C/O increased dyspnea orthopnea,bilateral lower-extremity swelling, 13 kg of wt gain in the past 3 weeks, and dietary nonadherence. LVEF 25%, NYHA class III, paroxysmal AF, and hyperlipidemia. BNP 2300 pg/mL, BUN 32 mg/dL, AST 40,ALT 42, INR 1.3. Medications include Coreg 12.5 mg BDI, Zestril 40 mg, Lasix80 mg BID, spironolactone 25 mg QD, and digoxin 0.125 mg QD After being IV loops, with only minimal UO,His Sao2 is now 87%(was 95%) on a 4-L nasal cannula, and an ABG is pending. His blood pressure is 110/75 mm Hg(was 108/62) and HR is 75BPM( was 82) beats/minute. The patient's SCr and K+ are 2.7 mg/dL(was2) and 5.4 mmol/L(was 4.9) respectively. In addition to a one-time dose of intravenous chlorothiazide, which regimen is most appropriate for this patient? A. NTG 20 mcg/min B. NTP 0.3 mg/kg/minute. C. Dobutamine 5 mcg/kg/minute. D. Milrinone 0.5 mcg/kg/minute.

A B=renal issues so NTG>NTP C=countered by BB use so poor choice here D=better agent but not ideal dosing scheme

66YOW(weight 70 kg) with a history of MI, HTN, hyperlipidemia, and DM, presents with sudden-onset diaphoresis, NV and dyspnea, followed by a bandlike upper chest pain (8/10) radiating to her left arm. She had felt well until 1 month ago, when she noticed her typical angina was occurring with less exertion. Her ECG reveals ST-segment depression and hyperdynamic T waves and positive cardiac enzymes. BP: 150/90 mm Hg, and all laboratory results are normal; SCr is 1.2 mg/dL. Home meds are ASA 81, Zocor 40, metoprolol 50 mg twice daily, and metformin 1 g twice daily. Which regimen is best for this patient? WHY??? A. Aspirin 325 mg, ticagrelor 180 mg one dose, and UFH 60-unit/kg bolus; then 12 units/kg/hour titrated to 50-70 seconds with an early invasive approach. B. Aspirin 325 mg and enoxaparin 70 mg subcutaneously twice daily with an early invasive approach. C. An ischemia-guided strategy with tirofiban 25 mcg/kg; then 0.15 mg/kg/minute plus enoxaparin 80 mg subcutaneously twice daily, aspirin 325 mg/day, and clopidogrel 300 mg one dose; then 75 mg once daily. D. An ischemia-guided strategy with aspirin 325 mg and ticagrelor 180 mg one dose; plus UFH 70-unit/kg bolus; then 15 units/

A high risk for future event thus ischemia guided(medical management) is not preferred and early invasive is EIS= DAPT to manage NSTE-ACS plavix, prasugrel and brillinta can be used in PCI cases UFH bolus is 60u/kg then 12u/kg/hr aPTT goal is 50-70 B= not DAPT so NO GO C=UFH> LMWH when used with GP2b/3a inihb D= wrong bolus then infusion rates. not ischemia guided in high risk

An 80-year-old woman had a total right knee replacement 3 days ago after conservative strategies for OA failed. Her medical history is significant for hypothyroidism, osteoporosis, OA, and hyperlipidemia. Her current medications include simvastatin 20 mg daily, risedronate 35 mg weekly, levothyroxine 75 mcg daily, and oxycodone/ acetaminophen 5/325 mg 1 tablet every 4 hours as needed for moderate pain. She is in the hospital preparing for discharge. As the pharmacist is counseling the patient on her discharge medication, the patient reports a new onset of "losing her water" the day before and again overnight. Which intervention would be most appropriate for this patient? A. Urinalysis. B. Pelvic floor exercises. C. Mirabegron 25 mg daily. D. Duloxetine 20 mg daily.

A infection is in DRIP so go there first B can be used if proven to have incontinence not just overactivity due to UTI C-urge incontinence or OAB once reversible causes ruled out D=stress incontinence tx

R.S. is a 48-year-old man (height 70 inches, weight 90 kg) who presents to the emergency department with pain and swelling in his left leg. On examination, his leg is warm to the touch and tender and has 3+ pitting edema below the knee. His D-dimer is positive, and his duplex ultrasonography identifies a femoral-popliteal DVT. He understands that he will need to receive anticoagulant therapy but wants to avoid any injections, if possible. He has good insurance coverage. His other medical conditions are hypertension, HIV, and dyslipidemia. His medications include benazepril 20 mg daily, ritonavir 100 mg daily, darunavir 800 mg daily, emtricitabine 200 mg/tenofovir disoproxil fumarate 300 mg daily, and atorvastatin 10 mg daily. His vital signs are stable, and his CrCl is 78 mL/minute/1.73 m2 . Which is the most appropriate anticoagulant regimen to initiate for R.S.? A. Rivaroxaban 15 mg twice daily for 21 days, followed by 20 mg daily. B. Edoxaban 60 mg daily. C. Warfarin 2.5 mg daily. D. Apixaban 5 mg twice daily for 7 days, followed by 2.5 mg twice daily.

A and D are the approved options D=best answer given DDI with Pgp and 3A4 inhib(ritonavir) A= this is normal dose but given interaction with Pgp and 3A4 inhib its use here is *Contraindicated* B= requires injectable med to load so nope C= needs load with Sq/IV therapy so nope

. Your pharmacy and therapeutics committee wants you to do a pharmacoeconomic analysis of a new drug available to treat decompensated HF. This drug has a unique mechanism of action. Unlike other available inotropic therapies that can increase mortality, this drug appears to reduce long-term mortality. However, its cost is 10-fold greater than other available drugs. Which pharmacoeconomic analysis would best determine whether this new drug is a better formulary choice than the currently available agents? A. Cost-minimization. B. Cost-effectiveness. C. Cost-benefit. D. Cost-utility.

B is new drug worth the cost with added mortality benefit A= CMA determines if therapeutically equivalant drug in class has same effect as drugs with lower cost C=determines if new program or service does enough to justify cost D=determines if a drug can improve QOL more than available therapies

J.W. is a 58-year-old man receiving enoxaparin 40 mg subcutaneously daily for VTE prophylaxis. While trying to shave himself at 6 p.m., he cuts his neck, and the medical team cannot stop the bleeding. His last enoxaparin dose was given at 8 a.m. Which would be the most appropriate protamine dose for J.W.? A. 10 mg. B. 20 mg. C. 40 mg. D. 50 mg.

B over 8 hours from dose of lovenox thus 1:1 become 1mg Enox: 0.5mg protamine

An order has been received for 2% sodium chloride. Assume no commercially available product is available. Using 0.9% sodium chloride and 23.4% sodium chloride, first determine how much of each is necessary to prepare 1 L of 2% sodium chloride. Second, calculate the osmolarity of 2% sodium chloride. Finally, determine whether the resultant solution should be administered through a central or peripheral intravenous infusion (molecular weight [MW] of sodium chloride is 58.5, osmotic coefficient is 0.93). A. Mix 951 mL of 0.9% sodium chloride plus 49 mL of 23.4% sodium chloride; osmolarity = 635 mOsm/L; peripheral intravenous infusion. B. Mix 951 mL of 0.9% sodium chloride plus 49 mL of 23.4% sodium chloride; osmolarity = 954 mOsm/L; central intravenous infusion. C. Mix 850 mL of 0.9% sodium chloride plus 150 mL of 23.4% sodium chloride; osmolarity = 954 mOsm/L; central intravenous infusion. D. Mix 850 mL of 0.9% sodium chloride plus 150 mL of 23.4% sodium chloride; osmolarity =513mOsm/L; peripheralintravenousinfusion

A. Use alligation If 0.9% sodium chloride contains 154 mEq/L, 2% should contain about 342 mEq/L. After completing the alligation, the correct amounts can be double-checked by verifying the amount of sodium chloride in the prepared product: 951 mL of 0.9% sodium chloride contains 146 mEq of sodium chloride, and 49 mL of 23.4% sodium chloride contains 196 mEq of sodium chloride; therefore, 146 mEq + 196 mEq = 342 mEq/L of sodium chloride in the final product. The osmolarity is calculated as (2 g/100 mL) × (1 mol/58.5 g) × (2 Osm/mol) × (1000 mOsm/Osm) × (1000 mL/L) × 0.93 = 635 mOsm/L Because of the osmotic coefficient (0.93), the sodium chloride does not completely dissociate in solution.

A 53-year-old Hispanic woman has a BMI of 44 kg/m2 and a history of gestational diabetes. Her mother and sister both have T2D. Two weeks ago, her A1C was 7.4%. Her fasting glucose concentration today is 178 mg/dL. She is asymptomatic. Which is the best course of action? A. Diagnose T2D and begin treatment. B. Diagnose T1D and begin treatment. C. Obtain another A1C today. D. Obtain another glucose concentration another day

A= 2 glycemic indicators present for DM B= nope C=one was done 2 weeks ago not needed again D=already 2 markers done so no need

An 81-year-old African American man (weight 90 kg) presents to the ED with chest pressure (10/10 on a pain scale). His ECG reveals ST-segment depression in the inferior leads. His medical history is significant for hypertension and chronic kidney disease. Pertinent laboratory results are troponin 5.8 ng/L, serum creatinine (SCr) 3.7 mg/dL, and estimated creatinine clearance (eCrCl) 20 mL/ minute. The patient has been given aspirin 325 mg single dose; a nitroglycerin drip, initiated at 5 mcg/minute, will be titrated to chest pain relief and blood pressure. The patient consents for cardiac catheterization after adequate hydration. Which anticoagulation strategy is most appropriate to initiate in this patient? A. Intravenous heparin 4000-unit intravenous bolus, followed by a 1000-unit/hour continuous infusion. B. Enoxaparin 90 mg subcutaneously every 12 hours. C. Fondaparinux 2.5 mg subcutaneously daily. D. Bivalirudin 67.5-mg bolus, followed by a 157- mg/hour infusion

A= 4k unit bolus is correct and infusion is limited to 1k units/hr C= fonda is not preferred in cases of NSTE- ACS invasive management B and D= would need to be dose adjusted given renal function to once daily. could be used if pt had good renal function

A 76-year-old woman (weight 47 kg) recently given a diagnosis of Hashimoto disease presents with mild symptoms of lethargy, weight gain, and intolerance to cold. Her thyroid-stimulating hormone (TSH) is 12.2 mIU/L and free thyroxine (T4 ) is below normal limits. She has a history of hypertension and underwent a coronary artery bypass surgery 2 years ago. Which would be the most appropriate initial treatment for this patient? A. Levothyroxine 25 mcg once daily. B. Levothyroxine 75 mcg once daily. C. Liothyronine 25 mcg once daily. D. Liothyronine 75 mcg once daily

A= Cardiac hx thus lower dose B=1.6mcg/days is common initial dose C and D= not first line

A 68-year-old man is admitted after an episode of syncope, with a presyncopal syndrome of seeing black spots and dizziness before passing out. Telemetry monitor showed sustained VT for 45 seconds. His medical history includes HF NYHA class III, LVEF 30%, two MIs, hypertension for 20 years, LV hypertrophy, DM, and diabetic nephropathy. His medications include lisinopril 5 mg/day, furosemide 20 mg twice daily, metoprolol 25 mg twice daily, digoxin 0.125 mg/day, glyburide 5 mg/day, atorvastatin 40 mg, and aspirin 81 mg/day. His blood pressure is 120/75 mm Hg, with heart rate 80 beats/minute, BUN 30 mg/dL, and SCr 2.2 mg/dL. Which is the best therapy to initiate for conversion of his sustained VT? A. Amiodarone 150 mg intravenously for 10 minutes, then 1 mg/minute for 6 hours, then 0.5 mg/minute. B. Sotalol 80 mg twice daily titrated to QTc of about 450 milliseconds. C. Dofetilide 500 mcg twice daily titrated to QTc of about 450 milliseconds. D. Procainamide 20 mg/minute, with a maximum of 17 mg/kg.

A= Correct - first line for VT pts with severe renal issues, HF and SHD B= Crcl is under 60 so sotalol needs renal adjustment. not good to cardiovert. better to prevent future issues C=only for AF not Vtach and cardioversion rates are low D= no procainamide (Class 1) if LV dysfunction

B.T. is 68-year-old man (height 72 inches, weight 96 kg) who recently underwent surgical aortic valve replacement with a bioprosthetic valve and is recovering well. He also has hypertension, dyslipidemia, systolic heart failure, and a history of sustained ventricular tachycardia. His current medications include carvedilol 25 mg twice daily, lisinopril 10 mg daily, furosemide 40 mg daily, amiodarone 400 mg daily, and atorvastatin 80 mg daily. His CrCl is 40 mL/minute/1.73 m2 , his hepatic function is normal, and his other laboratory data are within normal limits. Which is the most appropriate antithrombotic regimen for B.T.? FOR HOW LONG??? A. Aspirin 81 mg daily. B. Warfarin 7.5 mg daily to an international normalized ratio (INR) of 2.5-3.5. C. Apixaban 5 mg twice daily. D. Dabigatran 150 mg twice daily

A= bioprosthetic is low risk of clot so anticoag is not needed for the valve but CAN be used. Antiplatelet is adequate. INDEFINITE THERAPY B= warfarin is fine here(atleast 3 months and upto 6 months post surgery) but INR goal is 2-3. if it was a mechanical valve then its 2.5-3.5. ALSO initial dose is too high C and D= DOAC are not used unless pt has afib

B.D. is a 73-year-old man (height 69 inches, weight 80 kg) with newly diagnosed NVAF. He also has a history of hypertension, dyslipidemia, stable ischemic heart disease, and systolic heart failure. His current medications include aspirin 81 mg daily, enalapril 10 mg daily, atorvastatin 80 mg daily, metoprolol succinate 200 mg daily, furosemide 40 mg daily, spironolactone 25 mg daily, and amlodipine 10 mg daily. His current heart rate is 72 beats/ minute and blood pressure is 122/72 mm Hg. His laboratory values include potassium 4.9 mEq/L, stable SCr 1.9 mEq/L, and blood glucose 101 mg/dL. Six months later, B.D. needs to undergo percutaneous coronary intervention for management of his coronary artery disease. A. Rivaroxaban 10 mg daily, plus clopidogrel 75 mg daily. B. Apixaban 2.5 mg twice daily, plus clopidogrel 75 mg daily. C. Adjusted-dose warfarin to an INR of 2.0-3.0, plus aspirin 81 mg daily, plus clopidogrel 75 mg daily. D. Edoxaban 15 mg once daily, plus aspirin 81 mg daily, plus clopidogrel 75 mg daily

A= correct. lower risk of bleed vs warfarin. dose is reduced due to limited Crcl. would be 15mg/day if normal function B=5mg would be the dose C=DAPT and warfarin is okay but not optimal due to increased bleed risk vs Xarelto D= no data

L.S. is a 48-year-old woman with alcohol-induced cardiomyopathy. Her most recent LVEF is 20%; her daily activities are limited by dyspnea and fatigue (NYHA class III). Her medications include lisinopril 40 mg daily, furosemide 40 mg twice daily, carvedilol 12.5 mg twice daily, spironolactone 25 mg/day, and digoxin 0.125 mg/day. She has been stable on these doses for the past month. Her most recent laboratory results include sodium (Na) 140 mEq/L, potassium (K) 4.0 mEq/L, chloride 105 mEq/L, bicarbonate 26 mEq/L, blood urea nitrogen 12 mg/dL, SCr 0.8 mg/dL, glucose 98 mg/dL, calcium 9.0 mg/dL, phosphorus 2.8 mg/dL, magnesium 2.0 mEq/L, and digoxin 0.7 ng/mL. She weighs 69 kg, and her vital signs include BP 112/70 mm Hg and HR 72 beats/minute. She has normal breath sounds and no pedal edema. What is the best approach for maximizing the management of her HF? A. Increase carvedilol to 25 mg twice daily. B. Increase lisinopril to 80 mg/day. C. Increase spironolactone to 50 mg/day. D. Increase digoxin to 0.25 mg/day.

A= moved to goal dose B= at goal C=not needed D=inc tox without much benefit

A 53-year-old woman with a history of Graves disease had ablative therapy 3 years ago, after which she had significant symptom relief and became euthyroid. Her thyroid laboratory values today include TSH 0.12 mIU/L and free T4 3.8 g/dL. She states that many of her previous symptoms have returned but are mild. Which would be the most appropriate treatment for her condition? A. Methimazole. B. Lugol's solution. C. Propylthiouracil. D. Metoprolol.

A= once daily for hyperthyroid B= acute tx not chronic C=hepatotox risk and TID possibly D=symptoms not cause

A patient with newly diagnosed T2D is screened for diabetic nephropathy. The following laboratory values are obtained today: blood pressure 129/78 mm Hg, heart rate 78 beats/minute, urine albumin/creatinine 27 mg/g, and estimated CrCl 94 mL/minute/1.73 m2 . Which would be the most appropriate treatment strategy? A. No change in therapy is warranted. B. Add an angiotensin-converting enzyme (ACE) inhibitor. C. Add a thiazide-like diuretic. D. Reduce daily protein intake.

A= urine albumin/creatinine is under 30 so no need to add ACEi

A 66-year-old man is given a diagnosis today of T2D. Two weeks ago, his A1C was 7.5% and SCr was 1.8 mg/ dL (eGFR 25 mL/minute/1.73 m2 ). He has a history of hypertension, dyslipidemia, and systolic heart failure (New York Heart Association class III, ejection fraction 33%). He has 2+ pitting edema bilaterally. In addition to improvements in diet and exercise, which is best to initiate? A. Linagliptin. B. Pioglitazone. C. Exenatide. D. Metformin.

A=no renal function concerns B=hx of edema and HF nope C=renal impairment limits appropriateness of use here D= poor renal so nope

Which medication is most appropriate for a patient with a diagnosis of Cushing syndrome who has had inadequate symptom relief after surgical resection for a pituitary adenoma? A. Ketoconazole. B. Spironolactone. C. Hydrocortisone. D. Bromocriptine

A=reduces cortisol production B=hyper aldosteronism not overactive pituitary C=cushings is too much ACTH and cortisol so adding CS is not advisable D=treats acromegaly NOT cushings

if 24 of 310 pts getting dronedarond died of CV death and 9 of 317 pts in a placebo group died of CV death, whats the absolute risk of CV death in the dronedarone group? RR?

AR=0.077 AR of CV death with dronedarone/ total pts in dronedarone group 24/310=0.77419 RR=2.7 rate in Dronedarone group/ event rate in Placebo 24/310 / 9/317 2.72688

76YOW was recently admitted to a LTC facility for rehabilitation after several falls at home. Her medical history is significant for hypertension, hypothyroidism, Alzheimer disease (AD), hyperlipidemia, and osteoarthritis (OA) of the knee. She takes metoprolol succinate 50 mg daily, levothyroxine 75 mcg daily, atorvastatin 10 mg daily, and donepezil 10 mg daily. Her blood pressure is 126/80 mm Hg and heart rate is 66 beats/minute. Basic metabolic panel results are all within reference ranges; 25-hydroxyvitamin D concentration is 20 ng/mL, TSH is 1.89 mU/L, total cholesterol is 180 mg/dL, low-density lipoprotein cholesterol is 140 mg/dL, high-density lipoprotein cholesterol is 35 mg/dL, and triglycerides is 176 mg/dL. Her Mini-Mental State Examination (MMSE) score is 16/30, and her Geriatric Depression Scale score is 2/15. Which recommendation would be most appropriate to reduce this patient's risk of falls? A. Initiate memantine 5 mg daily. B. Initiate vitamin D 1000 units daily. C. Initiate aducanumab 1 mg/kg infusion every 4 weeks. D. Initiate calcium carbonate 500 mg twice daily.

B A-insufficient data presented to say namenda would help here C=only approved for MCI or mild AD. her MMSE score of 16 says they have MODERATE dementia and not qualify D=help with fractures not falls

ADR happens in hospital where pt needed additional care becaues of it. Which is most appropriate? A. A MedWatch form must be completed that explains the situation in which the ADR occurred. B. Institutions must create their own definition of ADR with which practitioners will be familiar. C. Hospital staff members must use the Naranjo algorithm to assess the severity of the ADR. D. Only severe or life-threatening ADRs need to be reported

B A=med watch is more so for unforseen ones not ones that are known risk B=TJC requires procedure for ADRs C=naranjo algorithm is for likely hood of cause and effect from drug induced event but not required

W.D. is a 55-year-old white female who was recently admitted to the hospital with acute myocardial infarction which was treated with a stent. She has a past medical history of HTN and GERD. She is visiting your clinic today for management of her cardiovascular medications. Her vitals today include BP 152/86 mm Hg and HR 82 beats/minute. Her labs are all WNL, including Na 140 mEq/L, K 4.3 mEq/L, and SCr 1.0 mg/dL. Her current medication regimen includes clopidogrel 75 mg daily, aspirin 81 mg daily, and atorvastatin 40 mg daily. What is the most appropriate approach to manage her HTN? A. Add carvedilol monotherapy B. Add lisinopril and metoprolol C. Add amlodipine and metoprolol D. Add lisinopril monotherapy

B A and D=2 drug therapy is best bc of BP being over 140 C= CCB is not recommended over BB or ACE/arb in this pt

61-year-old man comes to the emergency department with shortness of breath and bilateral lower leg edema. Pertinent vital signs and laboratory values include heart rate 30 beats/minute, blood pressure 102/57 mm Hg, K+ 7.9 mEq/L, Na+ 139 mEq/L, glucose 278 mg/dL, Ca2+ 8.8 mg/dL, digoxin 2.2 ng/mL, BUN 49 mg/dL, and SCr 2.4 mg/dL. His ECG shows wide QRS and peaked T waves. His medical history includes heart failure, atrial fibrillation, coronary artery disease, peripheral arterial disease, and diabetes. The patient has peripheral intravenous access and an external pacemaker. Which treatment is most appropriate? A. Calcium gluconate 10 mL intravenously over 2 minutes. B. Insulin 10 units intravenously. C. Sodium bicarbonate 50 mEq intravenously over 10 minutes. D. Albuterol 10 mg nebulized over 10 minutes

B A= is cautioned bc of dig DDi leading to bradycardia C=not well founded D= good ad on to insulin but not best option

J.M. is a 72-year-old woman (height 64 inches, weight 57 kg) who presents to the hospital with nonvalvular atrial fibrillation (NVAF). After her heart rate is controlled with metoprolol, she is asymptomatic. She also has hypertension, dyslipidemia, and depression. Her current medications include metoprolol 100 mg twice daily, enalapril 10 mg twice daily, and citalopram 20 mg daily. Her heart rate is currently 78 beats/minute and blood pressure is 134/86 mm Hg. Her serum creatinine (SCr) is 0.8 mg/dL and creatinine clearance (CrCl) is 60 mL/minute/1.73 m2 ; she has normal hepatic function. 1. Which best depicts J.M.'s CHA2 DS2 -VASc score and HAS-BLED score? A. CHA2 DS2 -VASc score of 1 and a HAS-BLED score of 1. B. CHA2 DS2 -VASc score of 3 and a HAS-BLED score of 1. C. CHA2 DS2 -VASc score of 5 and a HAS-BLED score of 4. D. CHA2 DS2 -VASc score of 3 and a HAS-BLED score of 2.

B CHF or LVEF<40 HTN Age >75=2 DM Stroke Vascular disease(MI) Age 65-74 Female HTN(>160) abnormal LFTs 1 or 2 Stroke hx bleeding Labile INR elderly(>65) drugs or alcohol use 1 or 2 pt has HTN hx but not currently, female and age of over 65 and under 75 then CHADVASC =3 HAS BLED =1

A 60-year-old man (weight 75 kg) presents to the ED with crushing substernal chest pain and ST-segment elevations on ECG. He has a medical history of diabetes and a 40 pack-year history of smoking. He is taken immediately to the catheterization laboratory for primary PCI, and a drugeluting stent is placed in his left anterior descending artery. In addition to aspirin, which regimen would best maintain this patient's stent patency? A. Clopidogrel 300-mg LD, followed by 75 mg daily for 12 months. B. Prasugrel 60-mg LD, followed by 10 mg daily for 12 months. C. Ticagrelor 180-mg LD, followed by 90 mg daily for 6 months. D. Clopidogrel 600-mg LD, followed by 75 mg daily for 6 months

B DAPT is for 12 months following ACS Prasugrel works faster than Plavix

A 75-year-old woman (weight 50 kg) is receiving PN after an extensive bowel resection. She is expected to require about 1 week of PN. She is receiving the following macronutrients in her formula: 70% dextrose 300 mL, 20% lipid 150 mL, and 10% AA 750 mL. 12. If these macronutrients are infused over 24 hours, which choice most closely approximates the total calories this patient is receiving daily? A. 20 kcal/kg. B. 26 kcal/kg. C. 30 kcal/kg. D. 35 kcal/kg.

B Dex= 3.4kg/g=>210x3.4=>714 Lipids=10kcal/g=>30x10=300 AA=4kcal/g=>75x4=>300 =1314kcal per day/50kg=26.3

In the US Nurses' Health Study (NHS) cohort study, where they looked at association of regular aspirin use (≥two 325 mg tablets/week) and colorectal cancer in 82,911 women found (RR, 0.77; 95% CI, 0.67-0.88) over 20 years of follow-up. In an another analysis of the NHS, regular aspirin use, investigator also found (hazard ratio [HR]=0.72, 95% CI 0.56-0.92), what does this say about the mortality from colorectal cancer? How can this data best be interpreted? A Those who takes aspirin regularly have 72% lower risk of colorectal cancer B. Those who takes aspirin regularly have 28% reduction in death from colorectal cancer C. Those who takes aspirin regularly have 0.23% lower risk of colorectal cancer D Those who takes aspirin regularly have 77% lower risk of colorectal cancer

B Hazard ratio can be used to compare time-to-event data between 2 groups. In this case the time to event was 20 years-to-death from colorectal cancer and the groups were patients taking aspirin 325mg 2 or more times a week and patients that did not. With the HR being 0.72, the patients taking the aspirin were 0.72 times likely to die at any time during the 20 years. This means that aspirin had a 28% reduction in death from colorectal cancer, 1 - 0.72 = 0.28.

An 85-year-old man presents with pain from hip OA. He has hypertension, coronary artery disease, and BPH. For his OA, he has been taking acetaminophen 650 mg three times daily. He reports that acetaminophen helps but that the pain persists and limits his ability to walk. Which is the best next step for this patient? A. Change acetaminophen to celecoxib. B. Add hydrocodone. C. Change acetaminophen to ibuprofen. D. Add glucosamine

B- AGS say after APAP use opioid for OA A and C are not gonna do enough benefit to outweigh risk of using NSAID on elerderly D= may help but not immediately

J.M. is a 72-year-old woman (height 64 inches, weight 57 kg) who presents to the hospital with nonvalvular atrial fibrillation (NVAF). After her heart rate is controlled with metoprolol, she is asymptomatic. She also has hypertension, dyslipidemia, and depression. Her current medications include metoprolol 100 mg twice daily, enalapril 10 mg twice daily, and citalopram 20 mg daily. Her heart rate is currently 78 beats/minute and blood pressure is 134/86 mm Hg. Her serum creatinine (SCr) is 0.8 mg/dL and creatinine clearance (CrCl) is 60 mL/minute/1.73 m2 ; she has normal hepatic function HAS BLED =1 CHADVASC =3 Which is the most appropriate stroke prevention strategy for J.M.? A. Aspirin 325 mg once daily. B. Rivaroxaban 20 mg once daily. C. Apixaban 2.5 mg twice daily. D. Edoxaban 30 mg once daily.

B= Crcl >50 so no need to dose adjust thus full dose 20mg daily is fine A=CHADVASC =3 so anticoag not platelet C=Scr is under 1.5, under 80. IS UNDER 60kg. needs 2 of 3. So pt would get 5mg BID not 2.5mg BID D=60mg daily is best

One year later, the patient has returned for complaints related to his BPH. His blood pressure is 118/74 mm Hg, heart rate 78 beats/minute, and PVR volume is 220 mL after voiding 150 mL. Current medications include doxazosin 4 mg daily, atenolol 25 mg daily, and amlodipine 10 mg daily. What is the most appropriate intervention for this patient? A. Initiate tadalafil 5 mg daily. B. Initiate finasteride 5 mg daily. C. Initiate alfuzosin 10 mg daily. D. Initiate saw palmetto supplement daily 40 mg injected into affected joint

B= avoids duplicate therapy and overlap in MOA. Good data supporting this combination in cases of BPH with urinary retention A= inc risk of Hypotension if given with non selective alpha blocker like Doxazosin C=selective(alf)+ non selective is not recommended D=not enough data backing this one

A 75-year-old woman with AD who lives at home with her husband has been treated with donepezil 10 mg daily for about 3 years. When she began therapy, her MMSE score was 21/30; her present MMSE score is 17/30. The patient cannot perform most IADLs but can perform most ADLs with cueing. About 2 months ago, her donepezil dose was increased to 23 mg, but she could not tolerate it, and it was reduced back to 10 mg daily. Her husband asks about changing her drug treatment to help maintain her function. Which is the next best course of action? A. Retry donepezil 23 mg daily. B. Initiate memantine 5 mg daily. C. Initiate aducanumab 1 mg/kg infusion every 4 weeks. D. Change donepezil to rivastigmine 9.5-mg patch daily.

B= benefit of them in combination A= retry is not useful if not tolerated previously C=not indicated for this level of dementia(moderate) D= changing agents is not best as pt did improve with donepezil at 10mg/day

J.T. is a 62-year-old man (height 72 inches, weight 85 kg) with a history of CHD (MI 3 years ago), HTN, depression, CKD (baseline SCr 2.8 mg/dL), PAD, osteoarthritis, hypothyroidism, and HF (LVEF of 25%). His medications include aspirin 81 mg/day, simvastatin 40 mg every night, enalapril 5 mg twice daily, metoprolol succinate 50 mg/day, furosemide 80 mg twice daily, cilostazol 100 mg twice daily, acetaminophen 650 mg four times daily, sertraline 100 mg/day, and levothyroxine 0.1 mg/day. His vital signs include BP 128/74 mm Hg and HR 72 beats/minute. Pertinent laboratory results include K 4.1 mEq/L, SCr 2.8 mg/dL, and a thyroid-stimulating hormone of 2.6 mIU/L. His HF is stable and considered NYHA class II. What is the best approach for maximizing the management of his HF? A. Discontinue metoprolol and begin carvedilol 12.5 mg twice daily. B. Increase enalapril to 10 mg twice daily. C. Add spironolactone 25 mg/day. D. Add digoxin 0.125 mg/day.

B= inc ACE to goal A=not consensus best move C=Scr is 2.5 so nope D=not on optimal therapy

A 70-year-old woman. She has complaints of fatigue, light-headedness, constipation, and "too many medicines." PMH: HTN, CAD(DES 8 years ago), COPD, DM, incontinence, frequent UTIs, depression, and moderate dementia. Vitals: BP 160/82 mm Hg, HR 51 beats/minute, RR: 16 O2 sat 99% on room air. Her current medications are as follows: Advair, ASA 81mg, APAP 650 TID, Plavix daily, donepezil 10 mg daily, glipizide 5 mg BID, lisinopril 10 mg daily, loratadine, metoprolol 50 mg BID, paroxetine 40 mg daily, ranitidine 150 mg BID, simvastatin 40 mg HS, and tolterodine 2 mg HS. Macrobid 100 mg twice daily for 10 days was initiated 3 days ago. Labs from 3 days ago: Na 130 mg/dL, CO2 24 mEq/dL, BUN 24 mg/dL, SCr 1.6 mg/dL, FBG 67 mg/dL, A1C 6.3%, UA: Positive for UTI Given the available patient information, which set of medications is least appropriate for this patient, according to the Medication Appropriateness Index? A. Advair, zantac, donepezil, detrol. B. toprol, plavix, zantac. C. Aspirin, glipizide, donepezil, macrobid. D. paxil, macrobid , zocor.

B=CORRECT. zantac has no indication for this pt,plavix is over the recommended tx time for her stent placement, metoprolol has DDI with donepezil A= advair is fine C= aspirin is fine D=macrobid is fine given hx of recurrent UTI

Regarding propylthiouracil and methimazole in the treatment of hyperthyroidism, which statement is most appropriate? A. Propylthiouracil is clinically superior to methimazole in efficacy. B. Propylthiouracil may be associated with greater liver toxicity than methimazole. C. Both agents are equally efficacious in the treatment of Hashimoto disease. D. Both medications should be administered three times daily

B=PTU has a BBW for hepatotox A= neither is better than the other C=hashimotos is HYPOthyroid D= methimazole is QD while PTU is upto to TID

T.J. is a 58-year-old African American woman presenting for routine follow-up of her chronic obstructive pulmonary disease. She has no other medical history. Her blood pressure today (average of 2 readings) is 138/88 mm Hg. Her HR is 77 beats/minute. Her BP at her last visit was 138/88 mm Hg. Her current medications include tiotropium dry powder inhaler daily and an albuterol metered dose inhaler as needed. Her labs include Na 140 mEq/L, K 4.0 mEq/L, Cl 102 mEq/L, bicarbonate 28 mEq/L, blood urea nitrogen 14 mg/dL, and SCr 0.8 mg/dL. Her 10-year ASCVD risk is 12.0%. What is the best approach for managing her HTN? A. Begin diet and lifestyle modifications only B. Begin lifestyle modifications and add amlodipine 5 mg daily C. Begin lifestyle modifications and add lisinopril 2.5 mg daily D. Begin lifestyle modifications and add lisinopril 2.5 mg daily plus hydrochlorothiazide 12.5 mg daily

B=stage 1 HTN with ASCVD risk so goal is 130/80 A= nope needs tx C=AA without HF, CKD or T2DM with albuminuria so HTN should be CCB/Thz D=1x drug is best for stage 1

J.O. is a 64-year-old woman with NYHA class II nonischemic dilated cardiomyopathy (LVEF of 30%). She presents to the heart failure (HF) clinic for a follow-up. She is euvolemic. Her medications include enalapril 10 mg twice daily, furosemide 40 mg twice daily, and potassium chloride 20 mEq twice daily. Her vital signs include BP 130/88 mm Hg and HR 78 beats/minute. Her laboratory results are within normal limits. What is the best way to manage J.O.'s HF? A. Continue current regimen. B. Increase enalapril to 20 mg twice daily. C. Initiate carvedilol 3.125 mg twice daily. D. Initiate digoxin 0.125 mg/day.

C A-needs optimal tx B- at target dose of Enalapril D- only if on optimal which pt is not on yet

A 62-year-old man presents to the emergency department (ED) with the chief concern of chest pain that woke him from sleep and radiates to his jaw. An electrocardiogram (ECG) reveals ST-segment depression in leads II, III, and aVF. His blood pressure is 112/62 mm Hg and heart rate is 60 beats/minute. Cardiac enzymes have been obtained, and the first troponin result was slightly positive. Preparations are under way to take the patient to the cardiac catheterization laboratory for evaluation. Which medication regimen is most appropriate for this patient at this time? A. Aspirin 325 mg, clopidogrel 600-mg loading dose (LD), and unfractionated heparin (UFH) infusion 80-unit/kg bolus, followed by 18 units/kg/hour and metoprolol 5 mg intravenously. B. Aspirin 81 mg; prasugrel 60-mg LD; UFH infusion 60-unit/kg bolus, followed by 12 units/kg/hour; and intravenous enalaprilat. C. Aspirin 325 mg, ticagrelor 180-mg LD, and UFH infusion 60-unit/kg bolus, followed by 12 units/kg/hour. D. Aspirin 81 mg, prasugrel 60-mg LD, nitroglycerin infusion at 10 mcg/minute, and bivalirudin 0.75-mg/kg bolus and 1.75-mg/kg/ hour infusion

C NSTE-ACS B and D- asa has to be 162-325mg A- UFH dose is 60u/kg then 12u/kg infusion A= IV metoporlol would be okay if HTN B= IV ACEi is not recommended for 24 hrs D=NTG drip is not best choice given low BP and HR

A 56-year-old white woman with a long history of hypertension because of nonadherence and recently diagnosed HF (EF 35%) presents to the local ED with blood pressure 210/120 mm Hg and heart rate 105 beats/minute. She states that she felt a little lightheaded but that she now feels okay. She ran out of her blood pressure medications (including hydrochlorothiazide, carvedilol, and lisinopril) 3 days ago. Her current laboratory values are within normal limits. Which medication is best for this patient? A. Sodium nitroprusside 0.25 mcg/kg/minute titrated to a 25% reduction in MAP. B. Labetalol 80 mg intravenously; repeat until blood pressure is less than 120/80 mm Hg. C. Resumption of home medications; refer for follow-up within 2 days. D. Resumption of home medications; initiate amlodipine 10 mg daily; refer for follow-up in 1 week.

C can use home meds bc this is urgency not crisis/emergency(end organ damage) A and B=IV meds in hospital not needed D= add an agent is not appropriate and follow up is too far off

A 70-year-old man is admitted to the hospital with peritonitis caused by severe inflammatory bowel disease. The patient has received adequate fluid resuscitation, and he is prescribed appropriate antibiotics. After several days of the patient being unable to tolerate oral or enteral nutrition, the physician consults the pharmacist to recommend a PN formula to be administered through a central line. The patient is hemodynamically stable, with normal electrolyte concentrations. Weight is 55 kg, BUN/SCr is 20/1.1 mg/dL, and WBC is 17 × 103 cells/mm3 . Assuming that appropriate electrolytes, multivitamins, and trace elements are included, which PN formula, when administered over 24 hours, will best provide this patient adequate calories, AAs, and lipids? A. AAs 10% 700 mL, dextrose 30% 325 mL, lipid 20% 500 mL. B. AAs 10% 450 mL, dextrose 70% 400 mL, lipid 20% 250 mL. C. AAs 10% 800 mL, dextrose 70% 350 mL, lipid 20% 250 mL. D. AAs 15% 900 mL, dextrose 50% 500 mL, lipid 20% 250 mL.

C 30 kcal/g of calories 1.5g/kg protein(AA) and 30% from lipids A-1000 calories as lipids B-0.8g/kg of AA D-too much AA

M.M. is a 63-year-old white woman who just finished 6 months of diet and exercise for dyslipidemia. She has a history of hypertension, DM, and asthma. She smokes one pack of cigarettes and drinks three beers per day. Her mother had HTN and suffered an MI at age 42 years. Her father had HTN and DM. Her medications are albuterol metered dose inhaler, lisinopril, metformin, linagliptin, and calcium carbonate antacids. Her vital signs include BP 134/84 mm Hg and HR 75 beats/minute. Her laboratory results are as follows: HDL-C 38 mg/dL, LDL-C 134 mg/dL, TG 186 mg/dL, TC 209 mg/dL, and hemoglobin A1C 8.6%. Her pooled cohort equation estimates a 10-year ASCVD risk of 27.8%. What is the most appropriate next step for M.M.? A. Initiate a low-intensity statin B. Initiate a moderate-intensity statin C. Initiate a high-intensity statin D. Initiate a high-intensity statin plus ezetimibe

C 40-75yrs with multiple ASCD risk factors(Smoking, low HDLC and HTN) so high intensity is best B would be correct if MM didnt have additional ASCVD risk factors A- low intensity is not recommended 1' treatment D-statin alone first then add on

A 70-year-old woman is admitted to the hospital with a broken arm after a fall. While in the hospital, she is on bedrest most of the time, loses 2 kg (current weight 63 kg), and has trouble sleeping. She is to be discharged to a rehabilitation facility for 2-3 weeks of therapy. Her medications at discharge are glipizide 5 mg daily, lisinopril 10 mg daily, aspirin 81 mg daily, a multivitamin daily, mirtazapine 15 mg at bedtime, calcium 500 mg twice daily, and tramadol 25 mg every 8 hours as needed for pain. To maintain and improve function in this patient, which intervention is best to implement? A. Add atorvastatin 10 mg daily. B. Increase lisinopril to 20 mg daily. C. Add vitamin D 1000 units twice daily. D. Change tramadol to naproxen 500 mg twice daily as needed for pain

C A and B are not needed D= naproxen increases risk of bleed and renal damage

patient (weight 65 kg) is receiving PN after abdominal surgery. The PN contains about 1600 kcal, including 100 g of protein, 500 kcal as lipid, and 200 g of dextrose. The following additives are also included in a 24-hour infusion of PN: sodium chloride 50 mEq, sodium acetate 100 mEq, potassium acetate 60 mEq, sodium phosphate 30 mmol, magnesium sulfate 12 mEq, calcium gluconate 10 mEq/day, multivitamins 10 mL, and trace elements 3 mL. The patient has an NG tube in place that is suctioning 400-500 mL/day, which is being replaced with an infusion of 0.9% sodium chloride. After 48 hours of PN, the patient has the following laboratory values: Na+ 140 mEq/L, K+ 3.8 mEq/L, Cl- 93 mEq/L, serum bicarbonate 35 mEq/L, pH 7.5, PCO2 47 mm Hg, and bicarbonate 36 mEq/L. Which adjustment to the PN formula is best at this time? A. Increase lipids to provide 750 kcal and reduce dextrose to 130 g. B. Increase sodium acetate to 150 mEq/day and discontinue sodium chloride. C. Increase sodium chloride to 150 mEq/day and discontinue sodium acetate. D. Add sodium bicarbonate 50 mEq to PN

C metabolic alk with compensatory resp acidosis low cl and high bicarb tx by replacing lost fluid with NS, use chloride over acetate salts B=could worsen m.alk A= hypercapnia is response to m. alk D=dont add sodium bicarb to TPN and not the tx for metabolic alk.

M.R. is a 51-year-old woman (Ht 65 inches, wt 98 kg, BMI 36 kg/m2 ) who presents to the ED with pain, swelling, and redness in her right leg up into her thigh. She also has some SOB and pain in the middle of her chest. She reports that she had a hysterectomy about 2 weeks ago and has not been moving around much at home in the past 2 weeks. On physical examination, her right leg is warmer than the left and tender to touch. Her cardiac examination appears normal, HR: 80 BPM, BP: 146/96 mm Hg, RR: 20B BPM, and O2 sat: 92% on RA. Her labs show positive D-dimer, a negative troponin, and CrCl of 65 . Ultrasound detects a right femoral-popliteal DVT, and CT reveals a PE. Her other conditions include HTN, DM, and dyslipidemia. She also reports smoking 1 PPD x30 years. Her current meds: lisinopril 10 mg QD, chlorthalidone 25mg QD, metformin 1000 mg BID, Pravachol 40mgQD, and Norco 5/500 Q6HPRN Which is the most appropriate treatment strategy for M.R.? A. Lovenox 100 mg Q12H and Pradaxa 150 mg BID x5 days then DC Lovenox B. Xarelto 15 mgBIDx 7 days, then 20 mg QD C. Enoxaparin 100 mg Q12H x 5 days then start edoxaban 60 mg daily. D. UFH 4kunit x1, then 1k units/hr and warfarin 7.5mg (2-3 goal)

C A= pradaxa itself is fine NO need to overlap therapies. just do 5 days of Enox THEN Pradaxa B=not 7 days, 21 days D=this is ACS UFH dosing. Also to high of start dose of INR and can use DOAC

A 69-year-old man is admitted to the hospital after a motorcycle collision. He had serious injuries resulting in a left leg above-the-knee amputation and has undergone several surgical procedures and rehabilitation in the past 2 weeks. His current medications include tamsulosin 0.4 mg daily, atenolol 25 mg daily, amlodipine 10 mg daily, senna/docusate 8.6/50 mg twice daily, oxycodone controlled release 10 mg every 12 hours, and hydromorphone 4 mg every 3 hours as needed for breakthrough pain (uses 1-2 daily). His blood pressure is 155/88 mm Hg, heart rate is 84 beats/minute, and postvoid residual (PVR) volume is 400 mL after voiding 110 mL. His chronic medical conditions are unremarkable except for hypertension, BPH, and gastroesophageal reflux disease. 9. Which intervention would be most appropriate for this patient? A. Change tamsulosin to alfuzosin 10 mg once daily. B. Increase atenolol to 50 mg daily. C. Change tamsulosin to doxazosin 1 mg daily. D. Reduce hydromorphone to 2 mg every 3 hours as needed for breakthrough pain

C A= same class no difference B= only helps BP not urinary retention C= would help with BP and bladder retention D= long acting opioid on so decreasing dose would do little here

P.M. recently had knee replacement surgery. She has normal renal and hepatic function. Which is the most appropriate regimen for preventing VTE in P.M.? A. Dabigatran 110 mg, followed by 220 mg once daily. B. Enoxaparin 40 mg subcutaneously once daily. C. Fondaparinux 2.5 mg subcutaneously once daily. D. Edoxaban 60 mg once daily.

C A=dose and not used in knee replacement B=Enox in Knee is 30mg BID D=not indicated for hip or knee in US

B.G. is a 62-year-old man (height 69 inches, weight 110 kg) hospitalized for a heart failure exacerbation. He has symptoms when doing only limited exertion and has been out of bed only to use the bathroom for the past 3 days. His medical history also includes stable ischemic heart disease, hypertension, type 2 diabetes, and a PE 2 years ago. He currently smokes 2 packs/day and drinks 1 glass of wine with dinner most evenings. His current medications include bisoprolol 5 mg daily, lisinopril 10 mg daily, aspirin 81 mg daily, ranolazine 1000 mg twice daily, furosemide 40 mg daily, spironolactone 25 mg daily, and metformin 850 mg twice daily. His blood pressure today is 110/70 mm Hg and heart rate is 58 beats/minute. His laboratory values are normal except for a brain natriuretic peptide of 1498 ng/mL. Which is the most appropriate VTE prevention strategy for B.G.? A. Give fondaparinux 5 mg subcutaneously daily. B. Give apixaban 2.5 mg orally twice daily. C. Give enoxaparin 40 mg subcutaneously daily. D. B.G.'s risk does not warrant prophylactic therapy.

C A=dose is 2.5mg B= not studied for long term prevention D= does have risk factors for DVT

A 65-year-old man (weight 80 kg) with a 3-day history of a body temperature of 102°F (38.9°C), lethargy, and productive cough is hospitalized for community-acquired pneumonia. His medical history includes uncontrolled hypertension and coronary artery disease. His vital signs include heart rate 104 beats/minute, blood pressure 112/68 mm Hg, and body temperature 101.4°F (38.6°C). His urine output is 10 mL/hour, K 4 mEq/L, BUN is 46 mg/dL, SCr is 1.7 mg/dL, and WBC is 10.4 × 103 cells/mm3 . Other laboratory values are normal. After 2 days of appropriate antibiotic treatment, the patient has a WBC of 9 × 103 cells/mm3 , and he is afebrile. His blood pressure is 135/85 mm Hg, and his urine output is 45 mL/hour. His albumin is 3.2 g/dL, BUN is 14 mg/dL, and SCr is 1.4 mg/dL. All other laboratory values are normal. His appetite is still poor, and he is not taking adequate fluids. He has peripheral intravenous access. Which option is most appropriate to initiate? A. Peripheral PN to infuse at 110 mL/hour. B. Albumin 5% 500 mL intravenously over 60 minutes. C. D5 W/0.45% sodium chloride plus potassium chloride 20 mEq/L to infuse at 110 mL/hour. D. Lactated Ringer solution to infuse at 75 mL/

C does not need active fluid resus anymore so now its maintenance intravenous fluids to prevent dehydration and electrolyte imbalances. This is typically accomplished by a combination of free water and 0.45% sodium chloride with K+ (Answer C). The infusion rate is calculated as 1500 mL + (60 kg × 20 mL/kg) = 2700 mL/24 hours, or about 110 mL/ hour.

A 68-year-old woman (weight 60 kg) is admitted to the hospital after a cardioembolic stroke. Her medical history is significant for Afib , acute MI, and diabetes. She has been unconscious for 48 hours. The medical team decides to start providing nutrition. All of her laboratory values, including glucose concentrations, are normal. Although she currently has no enteral access, she does have a peripheral IV. Which nutritional regimen is best for this patient? A. Insert a Central line and initiate PN containing 60 g of amino acids (AAs), 250 mL of 20% lipid emulsion, 300 g of dextrose, standard electrolytes, MVI, and trace elements in a volume of 2000 mL administered over 24 hrs. B. Insert a Central line and initiate PN containing 40 g of AAs, 250 mL of 20% lipid emulsion, 200 g of dextrose, standard electrolytes, MVI , and trace elements in a total volume of 2000 mL administered over 24 hrs. C. Insert an NG or ND feeding tube and infuse an isotonic formula (1 kcal/mL) starting at 25 mL/hr and advance to a goal rate of 65 mL/hr D. Insert a percutaneous endoscopic gastrostomy feeding tube and infuse an isotonic formula (1 kcal/mL) starting at 25 mL/hr and advance to a goal rate 100ml/hr

C gi system is working so use that! D-require surgery and is saved for long term use A and B- not preferred as GI system is working. B would be the preferred IV solution

60-year-old woman with New York Heart Association (NYHA) class IV heart failure (HF) (heart failure with reduced ejection fraction [HFrEF]) is admitted for increased shortness of breath and dyspnea at rest. Her extremities appear well perfused, but she has 3+ pitting edema in her lower extremities. Her vital signs include blood pressure 125/70 mm Hg, heart rate 92 beats/ minute, and oxygen saturation (Sao2 ) 89% on 100% facemask. After initiating an intravenous diuretic, which intravenous agent is best to rapidly treat this patient's pulmonary symptoms? A. Dobutamine. B. Milrinone. C. Nitroglycerin. D. Metoprolol

C pt is warm and wet vasodilator-esk effects are preferred here if BP is okay(if BP is not, THEN should use Inotropes) A and B increase CO which is not an issue here D= negative inotropic effects. NOT EUVOLEMIC SO NO GO

A female patient (weight 80 kg) in the intensive care unit has developed acute kidney injury caused by sepsis, and she requires intermittent hemodialysis daily to maintain her BUN/SCr ratio at 49:2.5 mg/ dL. Currently, she is receiving appropriate antibiotics and is hemodynamically stable. She has also been receiving PN providing 72 g of AAs per day. What is the best recommendation for this patient's protein intake? A. Reduce AAs to 40 g/day. B. Reduce AAs to 64 g/day. C. Increase AAs to 96 g/day. D. Increase AAs to 160 g/day

C- needs 1.2-1.5g/kg of protein A and B= does not need reduced protein D= too much(closer to 2 g/kg)

T.G. (weight 125 kg) is admitted for the treatment of a DVT with a PE. He is given a 10,000-unit bolus of UFH and initiated on an infusion of 2250 units/ hour. Twelve hours into the infusion, he begins to vomit blood. Which is the most appropriate protamine dose for T.G.? A. 100 mg. B. 50 mg. C. 39 mg. D. 22.5 mg.

C= For UFH protamine dosing its: dose in past hr(2250) 1/2 dose in previous hr(1125) 1/4 dose hr before that(563) totaled is 3938 in past 3 hours=39 mg protamine A= bolus was over 3 hours ago or else dose would be 100mg based on that but no more than 50mg is given at a time so NOPE B= no more than 50mg/dose D=only calculates for amount in past hr not 3 hrs

A patient (weight 70 kg) receives propofol at 45 mcg/kg/minute. Propofol is available at a concentration of 10 mg/mL and is mixed in a 10% lipid emulsion. Assuming the patient is receiving this infusion rate for 24 hours, which best approximates the total calories provided by the propofol infusion in a 24-hour period? A. 200 kcal. B. 250 kcal. C. 300 kcal. D. 500 kcal.

D 454ml/day 1.1 kcal/mL 500kcal/day

A 72-year-old man is admitted to the hospital for HF decompensation. The patient has progressively increased dyspnea when walking (now 10 ft [3 m], previously 30 ft [6 m]) and orthopnea (now four pillows, previously two pillows), increased bilateral lower-extremity swelling (3+), 13 kg of weight gain in the past 3 weeks, and dietary nonadherence. He has a history of idiopathic dilated cardiomyopathy (LVEF 25%, NYHA class III), paroxysmal AF, and hyperlipidemia. Pertinent laboratory values are as follows: BNP 2300 pg/mL (0-50 pg/mL), K+ 4.9 mEq/L, BUN 32 mg/dL, SCr 2.0 mg/dL (baseline 1.9 mg/dL), aspartate aminotransferase (AST) 40 IU/L, alanine aminotransferase 42 IU/L, INR 1.3, aPTT 42 seconds, blood pressure 108/62 mm Hg, heart rate 82 beats/minute, and Sao2 95%. Home medications include carvedilol 12.5 mg twice daily, lisinopril 40 mg/day, furosemide 80 mg twice daily, spironolactone 25 mg/day, and digoxin 0.125 mg/day. Which regimen is best for treating his ADHF? A. Carvedilol 25 mg twice daily. B. Sodium nitroprusside 0.1 mcg/kg/min IV. C. Furosemide 120 mg intravenously twice daily. D. Milrinone 0.5 mcg/kg/minute

C= preferred to get fluid off A= no bc not euvolemic so could make HF worse B and D=BP is okay so can use diuretic before inotrope. Also these are used in cold and wet(no perfusion) not warm and wet

M.R. is a 51-year-old woman (height 65 inches, weight 98 kg, body mass index [BMI] 36 kg/m2 ) who presents to the ED with pain, swelling, and redness in her right leg up into her thigh. She also has some SOB and pain in the middle of her chest. She reports that she had a hysterectomy about 2 weeks ago and has not been moving around much at home in the past 2 weeks. On physical examination, her right leg is warmer than the left and tender to touch. Her cardiac examination appears normal, with vital signs of heart rate of 80 beats/minute, blood pressure 146/96 mm Hg, respiratory rate 20 breaths/minute, and oxygen saturation 92% on room air. Her labs show positive D-dimer, a negative troponin, and CrCl of 65 . Ultrasound detects a right femoral-popliteal DVT, and CT reveals a PE. Her other conditions include HTN, DM, and dyslipidemia. She also reports smoking 1 PPD for the past 30 years. Her current meds: lisinopril 10 mg QD, chlorthalidone 25mg QD, metformin 1000 mg BID, Pravachol 40mgQD, and Norco 5/500 Q6HPRN How many risk factors for DVT does the pt have? A.3 B. 4 C.5 D. 6

C=5 Age over 40 obesity as BMI is over 30 immobility AND recent surgery- 2 separate lines smoking

A 70-year-old woman. She has complaints of fatigue, light-headedness, constipation, and "too many medicines." PMH: HTN, CAD(DES 8 years ago), COPD, DM, incontinence, frequent UTIs, depression, and moderate dementia. Vitals: BP 160/82 mm Hg, HR 51 beats/minute, RR: 16 O2 sat 99% on room air. Her current medications are as follows: Advair, ASA 81mg, APAP 650 TID, Plavix daily, donepezil 10 mg daily, glipizide 5 mg BID, lisinopril 10 mg daily, loratadine, metoprolol 50 mg BID, paroxetine 40 mg daily, ranitidine 150 mg BID, simvastatin 40 mg HS, and tolterodine 2 mg HS. Nitrofurantoin 100 mg twice daily for 10 days was initiated 3 days ago. Na 130 mg/dL, CO2 24 mEq/dL, BUN 24 mg/dL, SCr 1.6 mg/dL, FBG 67 mg/dL, A1C 6.3%, UA: Positive for UTI Which medications would best be discontinued, according to the Choosing Wisely criteria? A. Paxil, Zantac, donepezil, Detrol. B. Toprol, Plavix, Zantac. C. Glipizide, donepezil, Macrobid. D. Zantac, Macrobid, glipizide, Detrol.

C=A1c less than 7, donepezil needs reassessment of R>B. asymptomatic bacteruira is not indication for constant treatment AB and D= not addressed in this criteria antipsych, A1C goals BZD and sedative hypnotic use for mood issues(agitation, sleep, delirium) Cholinesterase inhib use(B>R) appetite stimulants for anorexia asymptomatic bacteruria tx???

H.D. is a 67-year-old man with a history of HTN and AF for 4 years. His medications include ramipril 5 mg twice daily, sotalol 120 mg twice daily, digoxin 0.125 mg/day, and warfarin 5 mg/day. He visits his primary care physician today after being discharged from the emergency department with increased fatigue on exertion, palpitations, and lower extremity edema. His vital signs today include BP 115/70 mm Hg and HR 88 beats/minute, and all laboratory results are within normal limits; however, his lower extremity edema has worsened. His INR is 2.8. His ECG shows AF. An echocardiogram reveals an LVEF of 35%-40%. H.D.'s physician would like to continue a rhythm control approach. What is the best treatment option for managing his AF? A. Discontinue sotalol and begin metoprolol succinate 12.5 mg/day. B. Discontinue sotalol and begin dronedarone 400 mg twice daily. C. Discontinue sotalol and begin amiodarone 400 mg twice daily, tapering to goal dose of 200 mg/day for the next 6 weeks. D. Continue sotalol and add metoprolol tartrate 25 mg twice daily.

C=amiodarone and dofetilide are the ONLY antiarrythmics that can be given in HFrEF A and D= HFrEF so cannot get sotalol add metoprolol is reasonable but not until HF is controlled B= dronedarone not recommended in HF pts with recent decomkp

A 76-year-old male smoker (weight 61 kg) has a history of HTN, BPH, and lower back pain. Three weeks ago, he began to have substernal chest pain with exertion w/dyspnea, which radiated to both arms and was associated with nausea and diaphoresis which have inc inf freq to 4 or 5x/day; they are relieved with rest. He has never had an ECG. Today, he awoke with 7/10 chest pain and went to the ED of a rural community hospital 2 hours later. He was acutely dyspneic and had ongoing pain. Home medications are aspirin 81 mg/day for 2 months, doxazosin 2 mg/day, and ibuprofen 800 mg three times daily. Vital signs include heart rate 42BPM and blood pressure 104/48 mm Hg. Laboratory results include BUN 45 mg/dL, SCr 2.5 mg/ dL, and troponin 1.5 ng/L (normal value less than 0.1 ng/L). His ECG reveals a 3-mm ST-segment elevation. Aspirin, ticagrelor, and sublingual nitroglycerin were given in the ED. The nearest hospital with a catheterization laboratory facility is 2½ hours away. Which regimen is best? A. alteplase 15 units IV+ enoxaparin 30-mg IV bolus. B. ischemia-guided treatment of UFH 4k unit IVx1, 800u IV/hr C. tenecteplase 35 mg IV + UFH 4ku IV x1 then 800u/hr IV D. Tx for PCI

C=renal impairment and age limit excretion so using shorter half life tenecteplase and UFH is preferred over tPA and Enox over 120min from PCI so use lytics for STEMI he is under 6 hours from onset reperfusion is possible A= too old for bolus and tPA needs more dosing info B=UFh alone is inadequate given on going symptoms D=would not fit window for PCI

The mean decrease in Heart Rate after initiating a beta blocker XYZ in 90 patients was 24 beat per minute with standard error of 3.78 beats for minute. What would be the 95% confidence interval for the decrease in HR after initiating drug XYZ?

CI: mean+/- 1.96 x SEM 24 + (1.96 x 3.78) = 31.4 mmHg 24 - (1.96 x 3.78) = 16.6 mmHg

what is a dichotomous variable

Categorical variable that falls into one of two variables and NO MORE positive or negative yes or no

A 64-year-old woman presents to the ED with the chief concern of palpitations. Her medical history includes hypertension controlled with a diuretic and an inferior-wall MI 6 months ago. She is pale and diaphoretic but can respond to commands. The patient's laboratory values are within normal limits. Her vital signs include blood pressure 95/70 mm Hg and heart rate 145 beats/minute; telemetry shows sustained VT. Although initially unresponsive to β-blockers, the patient is successfully treated with lidocaine. Subsequent electrophysiologic testing reveals inducible VT, and sotalol 80 mg orally twice daily is prescribed. Two hours after the second dose, the patient's QTc is 520 milliseconds. Which regimen change would be most appropriate for this patient? A. Continue sotalol at 80 mg orally twice daily. B. Increase sotalol to 120 mg orally twice daily. C. Discontinue sotalol and initiate dofetilide 125 mcg orally twice daily. D. Discontinue sotalol and initiate amiodarone 400 mg orally three times daily.

D A and B= this is an ADR from sotalol that will not improve if continued or increased SO NOPE C=risk of TDP with sotalol= risk with dofetilide as they are both class 3 agents D=low risk of TDP

S.D. is a 62-year-old woman (height 65 inches, weight 80 kg) with a history of significant primary mitral regurgitation. Her echocardiogram reveals significant leaflet flaring that is not amendable to mitral valve repair. She also has a history of hypertension, dyslipidemia, and gout. Her current medications include lisinopril 10 mg daily, hydrochlorothiazide 25 mg daily, simvastatin 40 mg daily, and allopurinol 300 mg daily. Her heart rate is 68 beats/ minute and blood pressure is 128/74 mm Hg. Her CrCl is 68 mL/minute/1.73 m2 . She is scheduled to undergo valve replacement surgery and will receive a mechanical mitral valve. You are discussing the oral anticoagulation post-operative plan with S.D.'s team. Which is the optimal regimen for preventing thrombosis? A. Adjusted-dose warfarin to an INR goal of 2.5-3.5, plus aspirin 81 mg daily. B. Adjusted-dose warfarin to an INR goal of 2.0-3.0, plus aspirin 81 mg daily. C. Adjusted-dose warfarin achieves a goal INR of 2.0-3.0. D. Adjusted-dose warfarin to an INR goal of 2.5-3.5.

D no indication for ASA mitral valve has higher INR goal than atrial thus 2.5-3.5 is appropriate here

A 72-year-old woman (height 66 inches, weight 82 kg) whose medical history is significant for rheumatoid arthritis (RA), type 2 diabetes, gastroesophageal reflux disease, and hypothyroidism presents to the clinic with inflammation of the joints of the hands and stiffness lasting 1-2 hours in the morning. She is a smoker. Her current medications include pantoprazole 40 mg daily, metformin 850 mg twice daily, levothyroxine 100 mcg daily, folic acid 1 mg daily, methotrexate 12.5 mg weekly, naproxen 500 mg twice daily, calcium 600 mg twice daily, and vitamin D 1000 units twice daily. Her laboratory tests show a negative rheumatoid factor (RF) but positive anti-cyclic citrullinated peptides. The physician determines that this is a flare of moderate disease. Which would be the most appropriate intervention for maintenance treatment of this patient's RA? A. Change naproxen to prednisone 20 mg daily. B. Change methotrexate to 25 mg intramuscularly. C. Change methotrexate to leflunomide 20 mg daily. D. Add sulfasalazine 500 mg twice daily and hydroxychloroquine 400 mg daily

D pt has recurring symptoms, moderate activity and poor prognostic factor(anti-cyclic citrullinated peptides) should add sulfa and plaquenil to MTX to have 2x or event 3x DMARD therapy A= pred is good bridge not long term tx B=IM has not significant benefit over PO C=no benefit doing this switch

A 74-year-old woman (weight 50 kg) has been receiving isotonic tube feedings at 60 mL/hour for the past 8 days through her gastrostomy feeding tube. She recently had an ischemic stroke; she is responsive but is not able to communicate. Her serum sodium was 142 mg/dL on the day the isotonic formula was initiated, and it has risen steadily to 149, 156, and 159 mg/dL on days 3, 4, and 8, respectively, after the start of the tube feedings. What is the best treatment for her hypernatremia? A. Administer sterile water intravenously at 80 mL/hour. B. Administer D5 W intravenously at 80 mL/hour. C. Administer D5 W/0.225% sodium chloride intravenously at 80 mL/hour. D. Administer water by enteral feeding tube 200 mL every 6 hours.

D 1mL of water for each calorie given EN is preferred but if not D5W can be given IV NEVER USE STERILE WATER=CELL LYSIS AND DEATH

Results from a Meta-analysis where they looked at frequency of postoperative arterial fibrillation in patients on Ascorbic acid after cardiac surgery found odds ratio, 0.44 (95% CI, 0.32 to 0.61). How can you interpret this data? A. Ascorbic acid increased frequency of postoperative arterial fibrillation after cardiac surgery by 44% B. Ascorbic acid decreased frequency of postoperative arterial fibrillation after cardiac surgery by 44% C. There was no statistically significant difference in frequency of postoperative arterial fibrillation after cardiac surgery D. Ascorbic acid decreased frequency of postoperative arterial fibrillation after cardiac surgery by 56%

D 44% means the group was associated with an event happening 44% of the time, compared to 1 (an event happening 100% of the time if unexposed), therefore 100 - 44 = 56%, which is the reduction caused by the exposure. Exposure is the use of ascorbic acid.

An 80-year-old woman presents to your clinic accompanied by her daughter, who no longer feels comfortable leaving her mother alone because of her mother's "increasing forgetfulness." The patient's medical history is significant for type 2 diabetes, hypertension, coronary artery disease, congestive heart failure, and OA. She takes the following medications: acetaminophen 650 mg every 6 hours as needed for pain, lisinopril 20 mg daily, furosemide 20 mg daily, potassium chloride 20 mEq daily, carvedilol 12.5 mg twice daily, and glipizide 5 mg daily. Her MMSE score is 18/30. Blood tests obtained last week showed a normal basic metabolic panel, except for a fasting plasma glucose reading of 65 mg/dL. Her hemoglobin A1C (A1C) is 5.6%. A urinalysis is unremarkable. No nutritional deficiencies are noted. The patient's blood pressure is 130/80 mm Hg and heart rate is 60 beats/minute. She receives a diagnosis of AD. Which initial intervention would be most appropriate to help with this patient's cognitive function? A. Donepezil 10 mg daily. B. Galantamine extended release (ER) 24 mg daily. C. Memantine 10 mg twice daily. D. Rivastigmine patch 4.6 mg daily.

D A= would start lower B= starts at 8mg C= no benefit for memory D= best option left

A 66-year-old African American man (height 70 inches, weight 91 kg) with AF and CHD (non- ST-segment elevation MI and stent placement 3 years ago) presents with palpitations. Rate control therapy, including trials of β-blockers and nondihydropyridine calcium channel blockers, has been unsuccessful in controlling his symptoms. He currently takes metoprolol succinate 50 mg/day, aspirin 81 mg/day, atorvastatin 80 mg/day, lisinopril 5 mg/day, and warfarin 4 mg/day. His laboratory results show INR 2.2, potassium 4.8 mEq/L, SCr 1.2 mg/dL. His BP is 110/70 mm Hg, and his HR is 95 beats/minute. Which is the best antiarrhythmic therapy for him? A. Disopyramide. B. Flecainide. C. Propafenone. D. Sotalol.

D A=1a are not preferred to AF so B and C= CHD so no 1C (flecanide and propafenone are ruled out) sotalol is C3 and can be used in CHD with good renal function

A 75-year-old woman admitted for pneumonia has a history of several non-ST-segment elevation myocardial infarctions (NSTEMIs). She had an episode of sustained ventricular tachycardia (VT) during this hospitalization. Her corrected QT (QTc) interval was 380 milliseconds on the telemetry. Her left ventricular ejection fraction (LVEF) was found to be 25%. Her serum potassium and magnesium were 4.6 mEq/L and 2.2 mg/dL, respectively. Which intravenous agent is most appropriate for this patient's ventricular arrhythmias? A. Procainamide. B. Metoprolol. C. Magnesium. D. Amiodarone.

D A=LVEF has to be over 40% or else can worsen HF B= can be used for non sustained Vt and SVT in CAD C= IV mag is not needed bc levels are normal D=1st line for pts with LVEF <40

You are working on a review article about newer treatment strategies for hypertensive crises. You want to ensure that you retrieve all relevant clinical trials and related articles on your subject. Which comprehensive database is most appropriate to search to ensure that you have not missed key articles? A. International Pharmaceutical Abstracts. B. Iowa Drug Information Service. C. Clin-Alert. D. Excerpta Medica.

D A=abstracts from journals B=full text articles but new ones can take a while to access and comes out MONTHLY C=ADE and DI and medical legal issues D= is comprehensive database of 7k journals from 74 countries. recent articles take 10 days to post

B.W. is a 78-year-old man with a history of HTN, peripheral arterial disease (PAD), gastroesophageal reflux disease, and asymptomatic atrial fibrillation (AF) for the past month. His therapy includes aspirin 325 mg/day, lansoprazole 30 mg every night, atenolol 50 mg/day, lisinopril 10 mg/day, and atorvastatin 20 mg/day. His vital signs include BP 132/72 mm Hg and HR 68 beats/minute. Which is the best therapy for B.W. at this time? A. Add diltiazem and rivaroxaban. B. Add digoxin and increase lisinopril to 20 mg/ day. C. Discontinue atorvastatin and add warfarin. D. Add apixaban and decrease aspirin to 81 mg/day.

D CHA2DVASC =4=> PAD< HTN, Age over 75 so give anticoagulant no reason to stop atenolol given HR no reason to add rate control tx like digoxin or diltiazem at this time PAD= lipitor needed BP is okay so dont need to increase lisinopril

One year later, the patient returns to the clinic. She has moved in with her daughter. Lately, she wanders around the house continuously. She often changes clothes, cries out, and asks repetitive questions. Her current medication regimen includes rivastigmine 9.5 transdermal patch daily, which she has been taking for the past 6 months. Which would be most appropriate for this patient's new behavioral symptoms? A. Initiate olanzapine 5 mg daily. B. Initiate risperidone 0.5 mg twice daily. C. Initiate pimavanserin 34 mg daily. D. Change acetaminophen to 650 mg every 6 hours around-the-clock

D Dementia with change in mental state could b helped with pain management due to unmet need A andB= try others first then use antipsychotics C= not used unless parkinsons pt

The patient has received the PN formula for 3 days. Her blood glucose concentrations have ranged from 220 to 280 mg/dL. She has orders for the following sliding scale of regular insulin: blood glucose 200-250 mg/dL, give 2 units; blood glucose 251-300 mg/dL, give 4 units; and blood glucose 301-350 mg/dL, give 6 units. She has been receiving 14-16 units of insulin daily through the sliding-scale orders. Her medical history is significant for hypertension, diabetes, chronic obstructive pulmonary disease, and colon cancer. Treatment was recently initiated with methylprednisolone 60 mg intravenously every 6 hours for a chronic obstructive pulmonary disease exacerbation. Today, the dose will be reduced to 40 mg intravenously every 8 hours. What is the best recommendation for better control of this patient's blood glucose? A. Add insulin glargine 10-20 units/day to PN. B. Change 70% dextrose in PN to D5 W. C. Increase the sliding-scale insulin to 4 units for blood glucose 200-250 mg/dL, 8 units for blood glucose 251-300 mg/dL, and 12 units for blood glucose 301-350 mg/dL. D. Add neutral protamine Hagedorn insulin (NPH) 5 units subcutaneously every 12 hours.

D add scheduled to SS to help cut out use of SS which is NOT preferred as solo insulin therapy B=cuts calories from the PN so no go here C=cant keep it alone. not recommended A=LA insulin is not recommended to be added to PN. regular can be

J.C. is a 62-year-old man (70 inches,135 kg [1 month ago 143 kg]) with a hx of DM, CKD, CHD, and high TG that, in the past, has resulted in pancreatitis. His family history is significant for his father having CHD and hypertriglyceridemia. He is not a smoker, but admits drinking a 6-pack of beer daily. Pertinent laboratory findings include A1C of 11.6% and SCr= 2.6 mg/dL. He currently takes atorvastatin 40 mg every evening, aspirin 81 mg/day, metformin 1000 mg twice daily, olanzapine 10 mg/day, metoprolol tartrate 50 mg twice daily, and coenzymeQ10 200 mg/day. His fasting lipid profile is TC 402 mg/dL, LDL-C unable to calculate, HDL-C 48 mg/dL, and TG 1500 mg/dL. His other laboratory values are within normal limits. Which best describes potential secondary causes of elevated TG concentrations that should be considered in J.C.? A. Obesity, poorly controlled diabetes, olanzapine, metoprolol, coenzyme Q10. B. Alcohol consumption, poorly controlled diabetes, weight loss, β-blockers. C. Obesity, calcium channel blockers, hyperthyroidism, alcohol consumption. D. Alcohol consumption, obesity, poorly controlled diabetes, olanzapine, metoprolol

D coQ10 does not affect TG so A is wrong weight loss does not increase TG so not B CCbs or hyperthyroid can increase TG so C is wrong

A 68-year-old man is admitted to the hospital for worsening shortness of breath during the past 2 weeks caused by heart failure. His serum sodium concentration on admission was 123 mEq/L. Other abnormal laboratory values include brain natriuretic peptide of 850 pg/mL and SCr of 1.7 mg/ dL. Chest radiography is consistent with pulmonary edema. The patient weighs 85 kg on admission, which is up 3 kg from his baseline weight. The patient is not experiencing nausea, headache, or mental status changes. The physician orders 3% sodium chloride to treat the hyponatremia. Which recommendation is best? A. 3% sodium chloride is an appropriate choice because the hyponatremia is probably acute. B. A 250-mL bolus of 3% sodium chloride is appropriate if used in combination with furosemide to prevent volume overload. C. 3% sodium chloride is appropriate if the serum sodium does not increase more than 10 mEq/L in 24 hours. D. The risks of 3% sodium chloride outweigh the potential benefit for this patient.

D likely a chronic situation based on presentation seems like CHF- usually are asymptomatic can worsen volume overlaod improving Na does not improve outcomes in CHF pts

After 1 day of NS infusions to help increase fluid volume and Na, a pt's blood pressure is 122/80 mm Hg(was 86/50), and heart rate is 80 beats/minute. Her serum sodium is 120 mEq/L, and K+ is 3.2 mEq/L; she still feels tired. She is eating a regular diet. Her ECG is normal. Which is the best recommendation? A. D5 W/0.9% sodium chloride plus potassium chloride 40 mEq/L to infuse at 100 mL/hour. B. 0.9% sodium chloride infused at 100 mL/hour. C. 3% sodium chloride infused at 60 mL/hour. D. Potassium chloride 20 mEq by mouth every 6 hours for 4 doses

D low Na and K. in low K, both Na and K are in cells and keep low serum conc to maintain cell function. replenishing K will help push Na out as well thus fixing both problems No EKG issues wo PO is okay over IV(see in A) regular diet so B is not appropriate C= no serious symptoms of low Na

J.S. is a 43-year-old man with HTN who presents for an annual physical examination. His family history is significant for his father having HTN. His only medication is lisinopril 10 mg/day. His BP is 145/90 mm Hg. A fasting lipid profile shows TC 238 mg/dL, TG 95 mg/dL, LDL-C 176 mg/dL, and HDL-C 43 mg/dL. His calculated 10-year ASCVD risk according to the pooled cohort equation is 3.9%. Which best describes the next step for management in J.S.? A. Initiate high-intensity statin therapy. B. Initiate fenofibrate 130 mg/day. C. Initiate moderate-intensity statin therapy. D. Do not initiate statin therapy and reevaluate risk in 4-6 years.

D pt is 40-75 without ASCVD, severe lipid issues or DM 10 yr risk is low risk so no statin needed(A and C) TG are normal so B is wrong reassess in a few yrs

A 43-year-old male trauma patient (height 75 inches, weight 100 kg) was recently extubated and is receiving PN. His PN formula contains 35 kcal/kg, protein 1.2 g/kg, and dextrose infusing at 4.4 mg/kg/minute, and 25% of total calories as lipid. He has gradually developed symptoms of hypercapnia and has developed a respiratory acidosis. The medical team is considering strategies to correct this to avoid reintubation. Which change to the PN formula could best correct this situation? A. Change PN to EN and maintain current caloric goals. B. Reduce dextrose amount in PN to 3 mg/kg/minute and increase lipid to maintain current caloric goal. C. Change electrolytes to the acetate salt in the PN to correct the acid-base imbalance. D. Reduce the calories to 25 kcal/kg to prevent overfeeding.

D pt is being overfed. the dextrose is turning into CO2 and water thus too much dextrose=>acidity if we lower caloric intake through PN will lower dextrose thus lower acidosis and overfeeding goes away

A 44-year-old man has consistently high blood pressure (172/98 mm Hg today), despite his documented adherence to two maximal-dose blood pressure medications. He has frequent headaches, increased thirst, and fatigue. His urine free cortisol is 45 mcg/24 hours (normal range 20-90) and plasma aldosterone/renin ratio is 125 (normal is less than 25). Which most likely caused this patient's uncontrolled hypertension? A. Cushing syndrome. B. Addison disease. C. Hyperprolactinemia. D. Hyperaldosteronism

D ratio, high BP= hypealdosteronism. cushing can be 2nd to HTN as well. A=24hr return would have been high if cushings B=addisons is cortisol def C=prolatcin seems okay and aldosterone is high

62-year-old man presents to the ED after several hours of chest discomfort. His ECG reveals a 1- to 2-mm ST-segment elevation with positive troponins. He has also had increasing shortness of breath and lower-extremity swelling over the past 2-3 weeks. His medical history is significant for tobacco use for 40 years, chronic obstructive pulmonary disease, diabetes, and hypertension. His blood pressure is 102/76 mm Hg and heart rate is 111 beats/minute. He has rales in both lungs and 2-3+ pitting edema in his extremities. His echocardiogram reveals an ejection fraction (EF) of 25%. After primary percutaneous coronary intervention (PCI), he is transferred to the cardiac intensive care unit. Which best describes the acute use of β-blocker therapy in this patient? A. Give 12.5 mg of oral carvedilol within the first 24 hours. B. Give 5 mg of intravenous metoprolol at the bedside. C. Give 50 mg of oral metoprolol succinate at discharge. D. Give no β-blocker at this time.

D- re evaluate close to DC if he could get low dose BB best to avoid BB bc can cause HF decompensation given he is NOT euvolemic should try diuresis first then BB B= would further inc risk of cardiogenic shock A and C=doses are fairly aggresive for his EF of 25% and marginal BP

Which statement best describes thrombotic risk in patients with valve replacement surgery? A. Bioprosthetic valves carry a higher risk of thrombosis than mechanical valves. B. The highest risk of thrombosis with bioprosthetic valve placement is during the first year after surgery. C. All patients with mechanical heart valves require bridging therapy during invasive procedures. D. Valve replacement in the mitral position carries a higher risk of thrombosis than in the aortic position

D. higher risk so higher INR goal. larger size and slower flow= higher risk A= biopros is lower risk mechanical is more un-natural thus higher clot risk. ASA can be used for bioprosthetics when warfarin is needed if mechanical B=3 months not 1 yr C= not all cases need bridging(mech atrial with bileaflet design and no risk factors for clots)

A 52-year-old man (weight 100 kg) with a history of HTN and hypertriglyceridemia presents for cardiac catheterization. He has had 3 hours of crushing 10/10 substernal chest pain radiating to both arms that began while he was eating his lunch (seated), which is accompanied by nausea, diaphoresis, and shortness of breath. He usually can walk several miles without difficulty and smokes 1.5 packs/day of cigarettes. Home medications are lisinopril 2.5 mg/day and aspirin 81 mg daily. Current vital signs include HR: 68 bpm and BP 178/94 mm Hg. His ECG reveals a 3-mm ST-segment elevation in leads V2-V4, I, and aVL. Serum chemistry values are within normal limits. The first set of cardiac markers shows positive troponins, Which regimen is best for this patient's STEMI? A. Reperfusion PCI and stenting with tirofaban, clopidogrel and ASA B. Reperfusion with reteplase plus UFH, clopidogrel, and aspirin. C. Reperfusion with tenecteplase, enoxaparin, aspirin, ticagrelor and bivalirudin D. Reperfusion with primary PCI with stenting, prasugrel, aspirin, and bivalirudin

D= PCI is being used with DAPT and bival is anticoag PCI is preferred for reperfusion if possible which this facility has= A and D A= anticoag + antiplatelet is needed C=bival is not studed with lytics

P.M. is a 52-year-old man (height 70 inches, weight 116 kg) with a history of HTN and a transient ischemic attack 2 years ago. He visits his primary care doctor with the chief concern of several weeks of a "fluttering" feeling in his chest on occasion. He thinks the fluttering is nothing; however, his wife insists he have it checked. His current medications include metoprolol tartrate 50 mg twice daily and aspirin 81 mg/day. He is adherent to this regimen and has health insurance, but he does not like to make the 3-hour trip to his primary care provider. His laboratory data from his past visit were all within normal limits. His vital signs today include BP 130/78 mm Hg and HR 76 beats/minute. All laboratory values are within normal limits. An electrocardiogram (ECG) reveals an irregularly irregular rhythm, with no P waves, and a HR of 74 beats/minute. A diagnosis of AF is made. What is the best approach for managing his AF at this time? A. Begin digoxin 0.25 mg/day. B. Begin diltiazem CD 240 mg/day. C. Begin warfarin 5 mg/day and titrate to a goal INR of 2.5. D. Begin dabigatran 150 mg twice daily.

D= no INR monitoring so good to go. A and B=AV rate control is good but if further help is needed can increase dose of metoprolol C=high risk bc of HTN, and TIA CHADVASC is 3 so could be used. warfarin goal of 2.5 but f/u is issue and DOAC is preferred if possible

A 34-year-old woman has a BMI of 33 kg/m2 . With dietary changes, she has lost 0.9 kg (2 lb) in 6 months. She exercises regularly but cannot do more because she has two jobs and young children. Her medical history is significant for depression, T2D, and substance abuse. Her current medications include metformin 1000 mg twice daily, insulin glargine 24 units once daily, and sertraline 100 mg once daily. Her A1C is well controlled, but she occasionally has hypoglycemic episodes. She is most concerned about weight loss. Which would be the best recommendation to help her lose weight? A. Continue her diet and exercise routine; additional intervention is unwarranted. B. Initiate liraglutide 3 mg once daily. C. Initiate phentermine/topiramate 3.75/23 mg once daily. D. Initiate orlistat 120 mg three times daily with meals.

D=best option A= not enough B=given hypoglycemia already adding his for DM with good A1C is not advisable C= CS so may be habit formin

A.M. is a 32-year-old woman with type 1 DM and HTN. Her current medication regimen is as follows: ramipril 10 mg/day, chlorthalidone 25 mg/day, amlodipine 10 mg/day, ethinyl estradiol 20 mcg/norethindrone 1 mg daily (for the past 2 years), and insulin as directed. Her vital signs today include BP 145/83 mm Hg, repeated BP 145/81 mm Hg; HR 82 beats/minute; height 66 inches; weight 70 kg. A.M. would prefer not to take any more drugs, if possible. 10. Which option is the best clinical plan for A.M.? A. No change in therapy is currently warranted. B. Advise weight loss and recheck her BP in 3 months. C. Change chlorthalidone to hydrochlorothiazide. D. Discuss changing her contraceptive method.

D=estrogen can inc BP A= need to address BP B=weight loss can help but not likely to help vs BMI okay(?) C=Hctz=chlorthalidone in terms of effect

J.M. is a 65-year-old woman with a history of HTN and poor medication adherence who presents to her primary care physician with shortness of breath and markedly decreased exercise tolerance. An echocardiogram reveals an LVEF of 65%, with diastolic dysfunction. J.M.'s medications include extended-release nifedipine 90 mg/day and hydrochlorothiazide 25 mg/day. Her vital signs include BP 128/78 mm Hg and HR 98 beats/minute. Her lung fields are clear to auscultation, and there is no evidence of systemic congestion. Which is the best pharmacologic management for J.M.? what kind of HF? A. Discontinue extended-release nifedipine and initiate diltiazem 240 mg/day. B. Discontinue hydrochlorothiazide and initiate furosemide 40 mg twice daily. C. Initiate digoxin 0.125 mg/day. D. Add lisinopril 5 mg/day.

HFpEF A is best BB or non DHP CCB to slow HR Nifedipine can cause reflex tachy B- no symptoms of congestion C- no role in diastolic dysfunctn D-bc this is HFpEF not rEF, its only used if therapy is needed after HR is decreased

A 56-year-old female patient with type 2 diabetes mellitus is not adequately controlled on the current metformin dose of 1,000 mg twice daily. The patient's A1c was checked one week ago, and it was 7.9%. The patient's body mass index is 32 kg/m2, and she has no other significant medical history. Which medication should be added to the patient's regimen? Glipizide Lantus Liraglutide Januvia

Liraglutide- weight loss and DM

Which of the following are ordinal values(multiple answers) -NYHA Classes -Sex -Improvement (y/n) - Grade of breast cancer

NYHA and breast cancer

The rate that an outcome will occur given a particular exposure, compared to the rate of the outcome occurring in the absence of that exposure is definition of which of the following? Incidence Prevalance OR RR CI

RR

if 24 of 310 pts getting dronedarond died of CV death and 9 of 317 pts in a placebo group died of CV death, whats the relative risk increase of dronedarone theray?

RRI: 1- RR RR=24/310 / 9/317 2.72688 1- 2.72688= -1.72688 thus the relative risk increase is 173%

In a publication, observed mean weight of 200 patients was reported as 69.4 ±9.3 kg. If 9.3 kg is the SEM (standared error of mean), calculate the SD (standared deviation) and variance?

SEM = (SD) / sqrt (n) SD = standard deviation n = sample size Variance= SD^2 SEM x √n = SD 9.3 x √200 = 131.52 kg = SD SD2 (square) = variance131.522 = 17298kg = variance

In the US Nurses' Health Study (NHS) cohort study, where they looked at association of regular aspirin use (≥two 325 mg tablets/week) and colorectal cancer in 82,911 women found (RR, 0.77; 95% CI, 0.67-0.88) over 20 years of follow-up. What does this say about the mortality from colorectal cancer? those taking aspirin over 2x/wk have _________ risk of colorectal cancer. say in terms of % and decimal

a 23% lower risk 0.77x as likely to get colorectal cancer


Kaugnay na mga set ng pag-aaral

Ch. 47 Integumentary Dysfunction

View Set

Chapter 1 - Perspectives on Maternal and Child Health Care

View Set

COP3014 - Chapter 7 Multiple Choice

View Set

Probability: Simple and Compound Events

View Set

Chapter 8 Lifting and Moving Patients

View Set